[Search for users] [Overall Top Noters] [List of all Conferences] [Download this site]

Conference rusure::math

Title:Mathematics at DEC
Moderator:RUSURE::EDP
Created:Mon Feb 03 1986
Last Modified:Fri Jun 06 1997
Last Successful Update:Fri Jun 06 1997
Number of topics:2083
Total number of notes:14613

624.0. "Putnam Competition" by BEING::POSTPISCHIL (Always mount a scratch monkey.) Tue Dec 09 1986 23:52

Newsgroups: sci.math
Path: decwrl!decvax!ucbvax!cartan!brahms!larsen
Subject: This year's Putnam Exam
Posted: 8 Dec 86 21:47:03 GMT
Organization: Math Dept. UC Berkeley
 
 
	The following is a copy of this year's William Lowell Putnam 
Mathematical Competition:
 
 
Morning session (3 hours)
 
 
Problem A-1.
	Find, with explanation, the maximum value of f(x) = x^3 - 3x on
the set of real numbers x satisfying x^4 + 36 <= 13x^2.
 
Problem A-2.
	What is the units (i.e. rightmost) digit of [10^20000 / (10^100 + 3)]?
Here [x] is the greatest integer <= x.
 
Problem A-3.
	Evaluate sum(arccot(n^2 + n + 1)) as n runs over non-negative integers.
Here arccot(t) for t >= 0 denotes the number theta in the interval 
0 < theta < pi/2 with cot(theta) = t.
 
Problem A-4.
	A transversal of an n x n matrix A consists of n entries of A, no two
in the same row or column.  Let f(n) be the number of n x n matrices A
satisfying the following two conditions:
 
	a)  Each entry a(i,j) of A is in the set {-1, 0, 1}.
	b)  The sum of a transversal is the same for all transversals of A.
 
An example of such a matrix is
 
				[-1  0 -1 ]
			    A = [ 0  1  0 ].
				[ 0  1  0 ]
 
Determine with proof a formula for f(n) of the form
 
		f(n) = a(1)b(1)^n + a(2)b(2)^n + a(3)b(3)^n + a(4),
 
where the a(i)'s and b(i)'s are rational rational numbers.
 
Problem A-5.
	Suppose f1(x), f2(x), ..., fn(x) are functions of n real variables
x = (x1, ..., xn) with continuous second order partial derivatives everywhere
on R^n.  Suppose further that there are constants c(i,j) such that
 
		d fi   d fj
		---- - ---- = c(i, j) for all i, j between 1 and n.
		d xj   d xi
 
(Here d denotes partial derivative.)  Prove that there is a function g(x)
on R^n such that fi + dg / dxi is linear for all i, 1 <= i <= n.  (A linear
function is one of the form a0 + a1 x1 + a2 x2 + ... + an xn.)
 
Problem A-6.
	Let a(1), ..., a(n) be real numbers, and let b(1), ..., b(n) be
distinct positive integers.  Suppose there is a polynomial f(x) satisfying 
the identity
 
			(1 - x)^n f(x) = 1 + sum(a(i) x^b(i)),
 
where the sum is taken from 1 to n.  Find a simple expression (not
involving sums) for f(1) in terms of b(1), ..., b(n) (but independent of
a(1), ..., a(n).)
 
 
Afternoon session (3 hours)
 
 
Problem B-1.
	Inscribe a rectangle of base b and height h and an isosceles triangle
of base b in a circle of radius 1 in such a way that the base of the triangle
coincides with a base of the rectangle.  For what value of h do the rectangle
and triangle have equal areas.
 
Problem B-2.
	Prove that there are only a finite number of possibilities for the
ordered triple T = (x-y, y-z, z-x) where x, y, and z are complex numbers
satisfying the simultaneous equations
 
		x (x-1) + 2 y z = y (y-1) + 2 z x = z (z-1) + 2 x y,
 
and list all such triples T.
 
Problem B-3.
	Let Z[x] consist of all polynomials in x with integral coefficients.
For f and g in Z[x] and m a positive integer we write "f = g (mod m)" to
mean that f-g is an integral multiple of m.  Let n and p be positive integers
with p prime.  Given that f, g, h, r, and s are in Z[x] with rf + sg = 1 (mod p)
 
			r f + s g = 1 (mod p),
			f g = h (mod p),
 
prove there exist F and G in Z[x] with F = f (mod p), G = g (mod p), and
F G = h (mod p^n).
 
Problem B-4.
	For a positive real number r, let G(r) be the minimum value of
|r - sqrt(m^2 + 2n^2)|, for all integers m and n. Prove or disprove the
assertion that lim G(r) exists and is 0 as r ---> infinity.
 
Problem B-5.
	Let f(x, y, z) = x^2 + y^2 + z^2 + x y z.  Let p(x,y,z), q(x,y,z),
and r(x,y,z) be polynomials with real coefficients satisfying
 
	f(p(x,y,z), q(x,y,z), r(x,y,z)) = f(x, y, z).
 
Prove or disprove the assertion that the sequence p, q, r consists of some
permutation of (+-)x, (+-)y, (+-)z, where the number of minus signs is 0 or 2.
 
Problem B-6.
	Suppose A, B, C, D are n x n matrices with entries in a field F
satisfying the conditions that A T(B) and C T(D) are symmetric and
A T(D) - B T(C) = I, where T denotes transpose and I is the identity matrix.
Prove that T(A) D - T(C) B = I.
 
 
 
My apologies in advance for misprints and lapses of notational clarity.
 
			Michael Larsen @ brahms.berkeley.edu
T.RTitleUserPersonal
Name
DateLines
624.1Attempted solution for A-1SSDEVO::LARYWed Dec 10 1986 14:2515
Bypass this if you want to try it yourself...

   4      2                 2       2
  x  - 13x  + 36 <= 0 --> (x  - 4)(x  - 9) <= 0 --> (x-3)(x-2)(x+2)(x+3) <= 0

This will be true when any of the terms are 0 or when an odd number of them
are negative; i.e. the set of real numbers that satisfies the inequality is

			[-3,-2] U [2,3].

		   3	      2	
The derivative of x  - 3x = 3x  - 3 which is positive throughout both
subranges, so the maximum must be at an righthand endpoint; trying -2 and 3
shows the max is at 3 and it is 18.

624.2attempted solution of A-2SSDEVO::LARYWed Dec 10 1986 14:3623
Bypass this if you want to try it yourself...


    20000     100          19900            -100
  10     / (10    + 3) = 10      / (1 + 3*10    )

     19900         1    -100    2     -200    3     -300
 = 10      * (1 - 3 * 10     + 3  * 10     - 3  * 10     + ...

	       198     -19800    199     -19900    200     -20000
	... + 3    * 10       - 3    * 10       + 3    * 10       - ...)

		      100    199     200      100
 = (some integer) * 10    - 3    + (3    / (10    + 3))

				    200    100     100
Now, the last term is less than 1 (3    = 9    < 10   ), so the rightmost
digit of the quotient will be expressed as

		 199		     49
	nnnn0 - 3     = nnnn0 - ((81)   * 27) = nnnnn0 - mmmmm7 = 3


624.3Attempt at solving A-3SSDEVO::LARYWed Dec 10 1986 15:0623
Bypass this if you want to try it...


			  -1  2		  -1
Lemma: sigma(i=0 to n) cot  (i +i+1) = cot  (1/(n+1))

						  -1	     -1
Proof by induction: 1)	its true for n=0, i.e. cot  (1) = cot  (1)

2)	If its true for i=0 through n-1, then

			   -1  2	   -1           -1  2	
	sigma(i=0 to n) cot  (i +i+1) = cot  (1/n) + cot  (n +n+1)

	Using the formula cot(x+y) = (cot(x)cot(y)-1)/(cot(x)+cot(y))
	this yields the desired result and we have a lemma.

Therefore,
			   -1  2			     -1
 sigma(i=0 to infinity) cot  (i +i+1) = lim(n-->infinity) cot  (1/(n+1))

			= cot (0) = pi/2

624.4Attempt at solving B-1CACHE::MARSHALLhunting the snarkWed Dec 10 1986 18:1425
    finally, one I CAN solve! (it IS the easiest one of the lot)  
    
              .
             / \
        ............. ____
        |  /     \  |
        | /   +   \ |   h
    	|/_________\| ____

                    
    	"+" :== center of circle
    
    when the height of the triangle that sticks up above the rectangle
    is equal to "h", the two areas will be the same.
    
    thus h + (1/2)h = r = 1
    
    		h = 2/3
    
                                                   
                  /
                 (  ___
                  ) ///
                 /
    
624.5Attempted solution to A-4SSDEVO::LARYWed Dec 10 1986 20:2153
Bypass this if you want to try it yourself. Its not a very pretty
solution - is there a better one?


Since each transversal contains exactly one element from each row and column,
if you pick two elements a(i,j) and a(k,l) in a transversal and substitute
the "other corners" a(i,l) and a(k,j) for them you still have a transversal.
If the sum of every transversal in A is the same, it follows that

	a(i,j) + a(k,l) = a(i,l) + a(k,j) for all i,j,k and l

which implies a(i,j) - a(k,j) = a(i,l) - a(k,l), which implies that each
row of the matrix A differs from each other row, and specifically from the
first row, by a constant; i.e.

(1)	a(i,j) = a(1,j) + d(i) for all i and j.

Conversely, any matrix that obeys formula (1) will have the sum of all
its transversals equal to sigma(j=1 to n)a(1,j) + sigma(i=2 to n)d(i), and
is thus the kind of matrix we are looking for. So, the set of matrices we
are looking for is exactly the set of matrices that obey (1).

Now, how many matrices of order n with elements in {-1, 0, 1} satisfy (1)?

Well, there are 3^n possible "first rows" of such a matrix. They fall into
three categories:

(a) First rows where max(a(1,j))-min(a(1,j)) = 0 - there are three such first
	rows, namely all 0, all 1, and all -1.

(b) First rows where max(a(1,j))-min(a(1,j)) = 1 - there are 2^(n+1)-4 such
	first rows, namely all rows consisting of just {0,1} except for all 0
	and all 1 (2^n-2 of these), and all rows consisting of just {0, -1}
	except for all 0 and all -1 (another 2^n-2 of these).

(c) First rows where max(a(1,j))-min(a(1,j)) = 2 - all the remaining first
	rows are of this type, namely 3^n-2^(n+1)+1.

Now:
For first rows of type (a) there are 3 choices of d(i) for every other row
For first rows of type (b) there are 2 choices of d(i) for every other row
For first rows of type (c) there are 1 choices of d(i) for every other row

So, the total number of matrices of order n is:

3 * 3^(n-1)  +  (2^(n+1)-4) * 2^(n-1)  +  (3^n-2^(n+1)+1) * 1^(n-1)

which in the form needed by the problem is:

	2 * 3^n  +  1 * 4^n  +  (-4) * 2^n  +  1 * 1^n

Whew!

624.6Problem A2ENGINE::ROTHWed Dec 10 1986 20:5026
624.7Problem B1ENGINE::ROTHWed Dec 10 1986 20:5226
624.8Problem B-4VINO::JMUNZERWed Dec 10 1986 21:0458
    B-4 solution:
    
Show that G goes to zero by picking a particular m and n for each r.

 **  **  **  **  **  **  **  **  **  **  **  **  **  **  **  ** 

Let  H(r)  =  | r - sqrt(M^2 + 2*N^2) |

where   M = floor (r)
and     N = floor ( sqrt( (r-M)*M ) )

Then G(r) <= H(r), and H(r) ---> zero

(1)  Example:	r = 1234.5
		M = 1234
		sqrt (617) = 24.8
		N = 24
		sqrt (1234^2 + 2 * 24^2) = sqrt (1523908) = 1234.467
		H(1234.5) = | 1234.5 - 1234.467 | = .033

                | r^2 - M^2 - 2*N^2 |         | r^2 - M^2 - 2*N^2 |
(2)  H(r) =  ---------------------------  <  -----------------------
              | r + sqrt(M^2 + 2*N^2) |                 r

(3)  Work on the numerator:

	Let   X = r - M   and   Y = sqrt( (r-M)*M ) - N

	| r^2 - M^2 - 2*N^2 |

	=  | r^2 - (r-X)^2 - 2*(sqrt( (r-M)*M - Y)^2 |

	=  | r^2 - r^2 + 2*r*X - X^2 - 2*(sqrt( X*M - Y)^2 |

	=  | 2*r*X - X^2 - 2*( X*M - 2*Y*sqrt( X*M ) + Y^2 ) |

	=  | 2*r*X - X^2 - 2*X*M + 4*Y*sqrt( X*M ) - 2*Y^2 |

	=  | 2*r*X - X^2 - 2*X*(r-X) + 4*Y*sqrt( X*M ) - 2*Y^2 |

	=  | 2*r*X - X^2 - 2*r*X + 2*X^2 + 4*Y*sqrt( X*M ) - 2*Y^2 |

	=  | X^2 + 4*Y*sqrt( X*M ) - 2*Y^2 |

	<= | X^2 | + | 4*Y*sqrt( X*M ) | + | 2*Y^2 |

	<= 1  +  4 * sqrt (M)  +  2

	<= 4 * sqrt(r)  +  3

                  4 * sqrt(r)  +  3
(4)  So H(r)  <  -------------------
                          r

	which ---> zero as r ---> infinity.
    
    
    John
624.9after all that, what's the answer?CACHE::MARSHALLhunting the snarkWed Dec 10 1986 21:059
    re .7:
    
    So what is "h"? 
                                                   
                  /
                 (  ___
                  ) ///
                 /
    
624.10ENGINE::ROTHWed Dec 10 1986 21:455
    Re:             -< after all that, what's the answer? >-

    Oh well, maybe I'll learn to read someday.

    - Jim
624.11Problem B2ENGINE::ROTHWed Dec 10 1986 21:4737
624.12B-2CLT::GILBERTeager like a childWed Dec 10 1986 21:5529
Solution for B-2 follows


The simultaneous equations

		x (x-1) + 2 y z = y (y-1) + 2 z x = z (z-1) + 2 x y,

are equivalent to

		(x-y) (x+y-1-2z) = 0
		(y-z) (y+z-1-2x) = 0

The solutions to these are:

		((x = y) or (x+y = 1+2z)) and ((y = z) or (y+z = 1+2x))

Going through the 4 possibilities, the solutions are:

		x = y = z
		x = y, z = 1+x
		y = z, x = 1+y
		z = x, y = 1+z

In these cases, the ordered triple T = (x-y, y-z, z-x) takes values:

		( 0,  0,  0)
		( 0, -1,  1)
		( 1,  0, -1)
		(-1,  1,  0)
624.13SSDEVO::LARYWed Dec 10 1986 22:035
I agree with .12.

The (0,1,1) triple can't be right because the components
have to sum to (x-y)+(y-z)+(z-x) = 0.

624.14A-6CLT::GILBERTeager like a childThu Dec 11 1986 10:2612
I 'found' what appears to be a solution to problem A-6, but as yet have
no idea of why it turns out the way it does, or even whether it is the
general solution.  It follows the <FF>.


	Let a(1), ..., a(n) be real numbers, and let b(1), ..., b(n) be
distinct positive integers.  Suppose there is a polynomial f(x) satisfying 
the identity (1 - x)^n f(x) = 1 + sum(a(i) x^b(i)), where the sum is taken
from 1 to n.  Find a simple expression (not involving sums) for f(1) in
terms of b(1), ..., b(n) (but independent of a(1), ..., a(n).)

	f(1) = product(b(i)) / n!, where the product is taken from 1 to n.
624.15ENGINE::ROTHThu Dec 11 1986 11:2210
624.16Problem A3ENGINE::ROTHThu Dec 11 1986 12:2429
I can't resist trying yet another one.

    Instead of summing the angles we can consider instead the problem
    of rotating in the complex plane and recover the resulting angle.

    arccot(k^2+k+1) = the included angle of the complex number (k^2+k+1) + j.

    Form the product PROD (k>=0) [(k^2+k+1) + j] = (1+j)*(3+j)*(7+j)*...

    The first few partial products z[k] = x[k] + j y[k] are

	z[0] = 1 + j
	z[1] = 2 + j 4
	z[2] = 10 + j 30
	z[3] = 100 + j 400

    Now assume that in the k'th product y[k-1] = k*x[k+1]
    and write out the recurrance

	x[k] = x[k-1]*(k^2+k+1) - y[k-1] = x[k-1]*(k^2+k+1) - k*x[k-1]
	y[k] = y[k-1]*(k^2+k+1) + x[k-1] = k*x[k-1]*(k^2+k+1) + x[k-1]

	x[k] = x[k-1]*(k^2+1)
	y[k] = x[k-1]*(k^3+k^2+k+1)

    But k^3+k^2+k+1 = (k+1)*(k^2+1) and so y[k] = (k+1)*x[k], and the
    included angle must therefore approach pi/2 as k approaches oo.

    - Jim
624.17more on A-6SSDEVO::LARYThu Dec 11 1986 15:1140
>	f(1) = product(b(i)) / n!, where the product is taken from 1 to n.

Yeah, I suspected that as the answer by plugging in

		f(x) = 1 + x + x^2 + ... + x^n

which makes the a(i) and b(i) terms easier to figure.

One approach that looks real promising, but has me pretty well bogged down
in the details, is to make use of the following properties of polynomials
with repeated roots, which can be derived by expressing the polynomial in
product-of-roots form and differentiating it repeatedly:

1)	If a polynomial P has an n'th order repeated root r, then

			     2			n-1
		dP       ,  d P       ,  ...   d   P
	P(r),	---(r)	    ---(r)	       -----(r) are all zero
		dx	      2			 n-1
			    dx		       dx

2)	for that same polynomial P, the value of the reduced polynomial
	Q = P / (r-x)^n at r is given by

	    n	n
	(-1)   d P
	---- * ---(r)
		 n
	 n!    dx

In this case, property (1) with r=1 gives a set of n simultaneous equations
in the a(i) and b(i), which can be used to express the a(i) in terms of the
b(i), and property (2) gives the value of f(1).

The term prod(b(i)) showed up real early when I tried to solve the set of
equations, but the complexity quickly became more than I could afford.....

There must be an easier way!
							Richie

624.18A1,2,3,4,6 B1,2,3,4,5,6HERON::BUCHANANAndrew @vbo DTN 828-5805Fri Jul 21 1989 16:41433